Difference between revisions of "1968 AHSME Problems/Problem 13"

(Created page with "== Problem == If <math>m</math> and <math>n</math> are the roots of <math>x^2+mx+n=0 ,m \ne 0,n \ne 0</math>, then the sum of the roots is: <math>\text{(A) } -\frac{1}{2}\quad ...")
 
m (See also)
 
(2 intermediate revisions by 2 users not shown)
Line 10: Line 10:
  
 
== Solution ==
 
== Solution ==
<math>\fbox{}</math>
+
By Vieta's Theorem, <math>mn = n</math> and <math>-(m + n) = m</math>. Dividing the first equation by <math>n</math> gives <math>m = 1</math>. Multiplying the 2nd by -1 gives <math>m + n = -m</math>. The RHS is -1, so the answer is <math>\fbox{B}</math>
  
 
== See also ==
 
== See also ==
{{AHSME box|year=1968|num-b=12|num-a=14}}   
+
{{AHSME 35p box|year=1968|num-b=12|num-a=14}}   
  
 
[[Category: Introductory Algebra Problems]]
 
[[Category: Introductory Algebra Problems]]
 
{{MAA Notice}}
 
{{MAA Notice}}

Latest revision as of 01:52, 16 August 2023

Problem

If $m$ and $n$ are the roots of $x^2+mx+n=0 ,m \ne 0,n \ne 0$, then the sum of the roots is:

$\text{(A) } -\frac{1}{2}\quad \text{(B) } -1\quad \text{(C) } \frac{1}{2}\quad \text{(D) } 1\quad \text{(E) } \text{undetermined}$

Solution

By Vieta's Theorem, $mn = n$ and $-(m + n) = m$. Dividing the first equation by $n$ gives $m = 1$. Multiplying the 2nd by -1 gives $m + n = -m$. The RHS is -1, so the answer is $\fbox{B}$

See also

1968 AHSC (ProblemsAnswer KeyResources)
Preceded by
Problem 12
Followed by
Problem 14
1 2 3 4 5 6 7 8 9 10 11 12 13 14 15 16 17 18 19 20 21 22 23 24 25 26 27 28 29 30 31 32 33 34 35
All AHSME Problems and Solutions

The problems on this page are copyrighted by the Mathematical Association of America's American Mathematics Competitions. AMC logo.png